Answer:
6.5 kilometers per hour
Step-by-step explanation:
90 minutes is 1 1/2 hours.
to find the unit rate you would divide the distance by the time so you would divide 9.75 by 1.5 (1.5 is the decimal version of 1 1/2) which would give you 6.5 kilometers per hour
Suppose that you have just been hired at an annual salary of $85,000 and expect to receive an annual raise of 6% per year. What
will be the total amount of money you will have earned after 8 years?
Answer:
$125,800
Step-by-step explanation:
Amount (A) = ?
Principal (P) = $85,000
Rate (r) = 6%
Time (t) = 8 years
Simple interest formula;
A = P(1 + rt)
A = $85,000(1 + 0.48)
A = $125,800
Point B has coordinates (1,2). The x-coordinate of point A is -11. The distance between point A and point B is 15 units. What are the possible coordinates of point A?
Answer:
We know that the x-coordinate of point A is -11, and that the distance between point A and point B is 15 units. Let's call the y-coordinate of point A "y". Then we can use the distance formula to find the two possible values of y:
d = sqrt((x2 - x1)^2 + (y2 - y1)^2)
where (x1,y1) = (-11,y) is the coordinates of point A, and (x2,y2) = (1,2) is the coordinates of point B. Substituting the values, we get:
15 = sqrt((-11 - 1)^2 + (y - 2)^2)
15 = sqrt(144 + (y - 2)^2)
15^2 = 144 + (y - 2)^2
225 = 144 + (y - 2)^2
81 = (y - 2)^2
y - 2 = ±9
y = 2 ± 9
So the possible coordinates of point A are (-11,11) and (-11,-7).
A patient is taking 4ml of 250mg/5ml amoxicillin, how many mg is she taking for every 4ml
The given parameter states 250mg/5ml, this means for every 1 ml, the patient takes 50mg, hence for every 4ml the patient will take 200mg
Given data
Density of amoxicillin = 250mg/5ml amoxicillin
= 50mg/ml
Mass of amoxicillin the patient will take for each 4ml volume
Mass = Density*Volume
Mass = 50*4
Mass = 200mg
Learn more about density here: https://brainly.com/question/6838128
2) A test consists of eight true/false questions. A student who forgot to study guesses randomly on every question. What is the probability that the student answers at most six questions correctly?
Answer:
Step-by-step explanation:
Guss
A Gallup poll surveyed 3,731 randomly sampled US in April 2021, asking how they felt about requiring proof of COVID-19 vaccination for travel by airplane. The poll found that 57% said they would favor it. (Gallup, 2021b)
The number of people who were in favor of the survey regarding the vaccination is 2127 people.
How many people favored it?A percentage is a value or ratio that may be stated as a fraction of 100. If we need to calculate a percentage of a number, we should divide it's entirety and then multiply it by 100. The percentage therefore refers to a component per hundred. Per 100 is what the word percent means. It is represented by %.
From the information, the poll surveyed 3,731 randomly sampled US in April 2021, asking how they felt and 57% said they would favor it.
Therefore, the number of people who favor it will be:
= Percentage × People surveyed
= 57% × 3731
= 0.57 × 3731
= 2127
Therefore, the concept of multiplication was used to get the number of people who favored it.
Learn more about percentages on:
brainly.com/question/24304697
#SPJ1
Complete question
A Gallup poll surveyed 3,731 randomly sampled US in April 2021, asking how they felt about requiring proof of COVID-19 vaccination for travel by airplane. The poll found that 57% said they would favor it. How many people favor it?
3/4 ∙( 15/4 −3 1/2 )÷1 1/3
Answer:
9/64
Step-by-step explanation:
\(=0.75\times \frac{\left(3.75+-3.5\right)}{1.3}\\0.75\times \frac{\left(3.75+-3.5\right)}{1.3}\\=0.75\times \frac{0.25}{1.3}\\=0.75\times 0.1875(\left a.k.a\:\frac{3}{16}\right)\\\frac{\left(3\times 0.1875\right)}{4}\\\mathrm{\left(Clarification:\:The\:\:math\:\:above\:\:is\:\:a\:\:broken\:\:down\:version\:of\:the\:original\:problem.\right)}\\=\frac{0.5625\left(\frac{9}{16}\right) }{4}\\=0.1406\left(\frac{9}{64}\right)\)
Hope this helps!
Solve for x pleaseeee fast
Answer:
x = 76
Step-by-step explanation:
-4( x - 26 ) = -200
-4x + 104 = -200
-4x = -304
x = 76
Hopefully this helps!
Brainliest please?
the value of x is 76
\( - 4(x - 26) = - 200\)
\( - 4x + 104 = - 200\)
\(104 + 200 = 4x\)
\(304 = 4x\)
\( \frac{304}{4} = x\)
\(x = 76\)
Mia hired a moving company. The company charged $500 or its services, and Mia gives the movers a 16% tip.
Answer:
The company charged $500 for its services,and Mia gives the movers a 16% tip. Now, we can add the tip amount to the cost of the service to find the total amount Mia paid: Total amount = Cost of service + Tip amount = $500 + $80 = $580
Step-by-step explanation:
The quality control manager at a factory records the number of equipment breakdowns each day. Let the random variable Y represent the number of breakdowns in one day. The standard deviation of Y is 0.28. Which of the following is the best interpretation of the standard deviation? A. The number of breakdowns on a randomly selected day is expected to be 0.28. B. The number of breakdowns on a randomly selected day will be 0.28 away from the mean. C. The average number of breakdowns per day for a random sample of days is expected to be 0.28 D. On average, the number of breakdowns per day varies from the mean by about 0.28. E. The number of breakdowns per day for a random sample of days is expected to be 0.28 away from the mean.
The correct option is (D). On average, the number of breakdowns per day varies from the mean by about 0.28.
Because standard deviation represents the typical distance between each data point and the mean.
The variability in a dataset is condensed into a single value called the standard deviation (SD). It shows the typical separation between the mean and each data point. Smaller values show that the data points are more evenly distributed around the mean, indicating that the dataset's values are generally stable.The average distance between each data point and the mean is known as the mean absolute deviation of a dataset. It offers us a sense of how variable a dataset is.
A discrete random variable's variance and standard deviation can be seen as measurements of the variability of the values the random variable assumes throughout the course of an experiment's multiple trials.So, the correct answer is On average, the number of breakdowns per day varies from the mean by about 0.28.
For more such questions on standard deviation
https://brainly.com/question/23907081
#SPJ4
Which of the following transformations maps triangle JKL to triangle MNO? What is the relationship between the triangles? Reflect triangle JKL over the x-axis; triangle JKL and triangle MNO are similar Rotate triangle JKL 180° clockwise about the origin; triangle JKL and triangle MNO are congruent Translate triangle JKL left 1 unit and downward 10 units; triangle JKL and triangle MNO are congruent
Dilate triangle JKL by a scale factor of 2 from the origin; triangle JKL and triangle MNO are similar
Translate triangle JKL left 1 unit and downward 10 units; the triangles are congruent
Describing the transformations that map triangle JKL to triangle MNO?From the question, we have the following parameters that can be used in our computation:
Triangle JKL to triangle MNO
From the figure, we can see that translating the triangle JKL left 1 unit and downward 10 units would give triangle MNO
The above also implies that the triangles are congruent
This is because translation is a rigid transformation
Read more about transformation at
https://brainly.com/question/4289712
#SPJ1
1. Which of the following is equivalent to 6/10? 5/9 9/15 8/12 36/100
You need to purchase a cover for this pool. A company sells covers for
$2.25 per square foot. Based on the area of the pool, how much would
the cover cost? Enter a number only - no symbols.
30
18
18
6
Answer:
1, 215
Step-by-step explanation:
To calculate the cost of the cover, we need to know the area of the pool. Assuming that the pool is rectangular and has the dimensions given by the four numbers provided (30, 18, 18, 6), we can calculate the area as follows:
Area = length * width
Area = 30 * 18
Area = 540 square feet
Therefore, the cover for this pool would cost:
Cost = Area * Price per square foot
Cost = 540 * $2.25
Cost = $1,215
So the cover would cost $1,215.
if k(x) = 3x, then f'(x)=? A. x³Ln3 B. 3xLn3 C. 3x/Lnx D. 3/3xLn3
The correct option is B .solution of given problem with the help of integrating the given function is 3xLn3
what is integration and function ?The area under a curve in a given range can be calculated mathematically via integration. To locate the region between the curve and the x-axis, it is necessary to find a function's antiderivative and evaluate it twice.
A function is a rule that gives each input value a distinct output value. It can be compared to a machine that processes inputs into outputs in accordance with a predetermined rule or formula.
According to given informationTo find f'(x), we need to take the derivative of f(x), where f(x) is the antiderivative of k(x).
Since k(x) = 3x, we can find f(x) by integrating 3x with respect to x:
f(x) = ∫ 3x dx = 3/2 x² + C
where C is a constant of integration.
Now we can find f'(x) by taking the derivative of f(x):
f'(x) = d/dx (3/2 x² + C) = 3x
Therefore, the answer is (B) 3xLn3. Option (A) is incorrect because there is no natural logarithm term in the derivative of f(x). Option (C) is incorrect because the derivative of 3x is 3, not 3/Ln(x). Option (D) is incorrect because there is no x in the denominator of the natural logarithm term.
To know more about integration Visit:
brainly.com/question/31433890
#SPJ1
solve: s=4 + square rooted s+5
If I = prt, which equation is solved for t?
O 1-pr=t
O
1-P-1
I
pr
O 1+pr=t
The solution of the equation for t is t = i/pr
How to determine the equation for t?The equation is given as
i = prt
Divide both sides of the equation by p
So, we have the following equation
i/p = prt/p
Divide both sides of the equation by r
So, we have the following equation
i/pr = prt/pr
Evaluate the quotients
i/pr = t
Rewrite as
t = i/pr
By the above computation, we changed the subject of the formula in i = prt from i to t.
This implies that solving for t is a concept of subject of formula
Hence, the solution is t = i/pr
Read more about subject of formula at
https://brainly.com/question/10643782
#SPJ1
A group of ten office workers recorded the distance, in kilometres, they walked on Monday. Their results are listed below. (a) Find the mean distance, in kilometres, walked by this group of office workers on Monday. The following box-and-whisker plot represents the results listed above. (b) (i) Find the value of
p
. (ii) Find the interquartile range. Distances less than
r
kilometres walked on Monday are considered outliers. (c) Find the value of
r
. A group of twenty freelancers also recorded the distance, in kilometres, they walked during the same Monday. The mean distance walked by this group of freelancers on the Monday is
1.7 km
with a standard deviation of
1 km
. (d) (i) Find the total distance the group of freelancers walked on the Monday. (ii) Find the combined mean distance that all thirty office workers and freelancers walked on the Monday. (iii) On Tuesday, all twenty freelancers walked double the distance they walked on Monday. Calculate the variance in the distance walked by the freelancers on the Tuesday.
A) The mean distance walked by the group of office workers on Monday was approximately 6.89 km.
b) The interquartile range was approximately 2.2 km.
c) The value of r would be: r ≈ 4.1 km
d) (i) Total distance walked = 1.7 km * 20 = 34 km
(ii), the combined mean distance walked by all thirty workers was approximately 3.28 km.
(iii) the standard deviation is not given directly, we can use the fact that the standard deviation is equal to the square
(a) The mean distance walked by the group of ten office workers on Monday can be found by adding up all of their distances and dividing by the number of workers:
Mean = (5 + 2 + 3 + 6 + 5 + 8 + 5 + 7 + 6 + 4) / 10
= 51 / 10
= 5.1 km
(b) (i) The value of p is the median of the data set. Since there are 10 data points, the median is the average of the 5th and 6th smallest values:
p = (5 + 5) / 2
= 5 km
(c) The value of r represents the maximum distance that is not considered an outlier. From the box-and-whisker plot, we can see that the maximum value is 8 km, which is not an outlier. Therefore,
r = 8 km
(d) (i) The total distance walked by the group of 20 freelancers can be found by multiplying their mean distance by the number of freelancers:
Total distance = Mean distance * Number of freelancers
= 1.7 km * 20
= 34 km
(d)
(i) The total distance walked by the group of 20 freelancers can be found by multiplying their mean distance by the number of freelancers:
Total distance = Mean distance * Number of freelancers
= 1.7 km * 20
= 34 km
(ii) The combined mean distance that all 30 workers walked on Monday can be found by adding up the total distance walked by all workers and dividing by the total number of workers:
Combined mean distance = (Total distance by office workers + Total distance by freelancers) / Total number of workers
= (51 km + 34 km) / 30
= 85 / 30
= 2.83 km
(iii) On Tuesday, all 20 freelancers walked double the distance they walked on Monday. The variance in the distance walked by the freelancers on Tuesday can be found using the formula for the variance of a constant times a random variable:
Var(aX) = a^2 Var(X)
Since the freelancers walked double the distance, the variance of the distance they walked on Tuesday is:
Var(Tuesday distance) = 4 * Var(Monday distance) = 4 * (1 km)^2 = 4 km^2
Learn more about Mean
https://brainly.com/question/30112112
#SPJ4
A rectangle with a perimeter of 40 inches has a width that is 3 times the length. What is the length And the width
The Length is 5 inches and the Breadth is 15 inches
What is Perimeter?
A perimeter is a closed route that covers, surrounds, or outlines a two-dimensional form or length. The circumference of a circle or an ellipse is its perimeter. There are various practical applications for calculating the perimeter.
Solution:
Perimeter of Rectangle = 2*(Length + Breadth)
According to the question
Breadth = 3*Length
Perimeter = 40
40 = 2*(Length + 3*Length)
20 = 4*Length
Length = 5 inch
Breadth = 15 inch
To learn more about Perimeter from the given link
https://brainly.com/question/19819849
#SPJ1
d) How many three-digit numbers greater than 330 can be formed from the digits 0, 1, 2, 3, 4, 5, and 6?(
The question is incomplete. Here is the complete question.
(a) How many three-digit numbers can be formed from the digits 0,1,2,3,4,5 and 6, if each digit can be used only once?
(b) How many of these are odd numbers?
(c) How many are greater than 330?
Answer: (a) 180
(b) 75
(c) 105
Step-by-step explanation:
(a) In the group, there are 7 digits. A three-digit number can not start with zero, otherwise, it will be a 2-digit number. So:
For the hundreds position, there are 6 choices.
For the tens position, since the digit can be used only once, there are 6 choices.
For the unit position, there are 5 choices.
The total three-digit number formed is: 6*6*5 = 180
(b) To form an odd number, the unit position must be an odd digit, then:
unit position has 3 choices;
hundreds position has 5 choices;
tens position has 5 remaining choices.
The total three-digit odd number is: 3*5*5 = 75
(c) The number formed must be greater than 330, so:
If the number start with a 3, to be greater, there are 3 other choices (4, 5 and 6), so Tens position has 3 choices and Unit position has 5 choices.
Total number is: 3*5 = 15
Another possibility is the number starts with a digit bigger than 3 and so, there are 3 choices.
Tens position has 6 choices;
Unit position has 5 choices;
Total possibilities are: 3*6*5 = 90
The total number of ways a three-digit number is greater than 330 is:
90 + 15 = 105
Complete the following food-cost problems:
FC ÷ FC % = SP 2.75 ÷ 25% =
FC ÷ SP = FC % 3.80 ÷ $15.00 =
SP x FC % = FC 24.00 x 21% =
The following food-cost problems:
FC ÷ FC % = SP: 2.75 ÷ 25% = 11.
FC ÷ SP = FC %: 3.80 ÷ $15.00 = 25.33%.
SP x FC % = FC: 24.00 x 21% = 5.04.
To complete the food-cost problems, we will apply the given formulas and solve for the missing values.
FC ÷ FC % = SP
Given: FC = 2.75, FC % = 25%
Substituting the values into the formula, we have:
2.75 ÷ 25% = SP
To divide by a percentage, we convert the percentage to a decimal by dividing it by 100:
2.75 ÷ 0.25 = SP
Simplifying the division, we get:
11 = SP
Therefore, the selling price (SP) is 11.
FC ÷ SP = FC %.
Given: FC = 3.80, SP = $15.00
Substituting the values into the formula, we have:
3.80 ÷ $15.00 = FC %
To divide by a dollar amount, we can simply perform the division:
0.2533... = FC %
Therefore, the food cost percentage (FC %) is approximately 25.33%.
SP x FC % = FC
Given: SP = 24.00, FC % = 21%
Substituting the values into the formula, we have:
24.00 x 21% = FC
To multiply by a percentage, we convert the percentage to a decimal by dividing it by 100:
24.00 x 0.21 = FC
Simplifying the multiplication, we get:
5.04 = FC
Therefore, the food cost (FC) is 5.04.
In summary:
2.75 ÷ 25% = 11
3.80 ÷ $15.00 = 25.33%
24.00 x 21% = 5.04
For similar question on selling price.
https://brainly.com/question/16710746
#SPJ8
Zev read 15% of his 56-page book. Kelly read 12% of her 64-page book. Marneisha read 14% of his 68-page book Aleisha read 10% of her 72-page book. what the grastest
Answer:
Marneisha with 9.52 pages.
Step-by-step explanation:
Zev: 56×0.15 = 8.4
Kelly: 64×0.12 = 7.68
Marneisha: 68×0.14 - 9.52
Aleisha: 72×0.10 = 7.2
11. The area of a circle is πr2 where r is the length of the radius. Thus, one could take the measure of the central angle in degrees and ____________ _________. Then, multiply that result by πr2.
The measure of the central angle θ in degrees and then multiply by 1/(2π). Then that result by πr²
What is the area of the sector?We will use the following points to find the area of the sector;
First of all, the angle that is formed in a full rotation is 2π.
The area of a circle is given by the formula; A = πr²
The central angle is given by the angle symbol θ. Thus, we have to multiply the area of the circle by a factor θ/2π
Thus, the area of a sector is given by the formula;
A = (θ/2π) * πr²
Read more area of sector at; https://brainly.com/question/15050805
#SPJ1
Evaluate the rational number expression and express the result in simplest form. 34 ÷ (12 · 38)
The rational number expression in simplest form is 17/ 228.
How to evaluate a given mathematical expression with variables if values of the variables are known?You can simply replace those variables with the value you know of them and then operate on those values to get a final value. This is the result of that expression at those values of the considered variables.
WE have the rational number expression is given as
34 ÷ (12 · 38)
Solving;
34 ÷ (12 · 38)
34 ÷ (456)
= 17 / 228
Therefore, the rational number expression in simplest form is 17/ 228.
Learn more about evaluating a function at a value here:
https://brainly.com/question/2753269
#SPJ1
\(2(4z - 2) = 44\)
can you help me
2(4z−2) = 44
Simplify both sides of the equation.
Distribute:
(2)(4z)+(2)(−2) = 44
8z + −4 = 44
8z − 4 = 44
Add 4 to both sides.
8z − 4 + 4 = 44 + 4
8z = 48
Divide both sides by 8
8z/8 48/8
z = 6
Answer:
z=6
Step-by-step explanation:
Write the addition equation as a multiplication equation.
8 + 8 +8= 24
Answer:
3 × 8 = 24
3 × 8 is the multiplication equation.
Answer:
8x=24
Step-by-step explanation:
Please sayb this was helpful
The number of salespeople assigned to work during a shift is apportioned based on the average number of customers during that shift. Apportion 16 salespeople using Jefferson's method given the information below.
Shift Morning Midday Afternoon Evening
Average number of customers 125 280 460 560
Salespeople to assign
What modified divisor did you use?
a) Using the standard divisors according to Jefferson's apportionment method, the number of salespeople assigned to work each shift is as follows:
Shift Morning Midday Afternoon Evening Total
Number of
salespeople assigned 1 3 5 7 16
b) The modified or adjusted divisor used represents the average number of customers for each shift divided by the standard divisor.
What is the standard divisor?The standard divisor shows the ratio of the total population to the number of seats.
The standard divisor gives an insight about the number of people each seat represents.
Standard divisor = Total number of customers / Total number of salespeople
The total number of customers = 1,425
The number of salespeople = 16
a) Standard divisor = 1,425 ÷ 16
= 89.0625
Shift Morning Midday Afternoon Evening Total
Average number of
customers 125 280 460 560 1,425
Standard divisor 89.0625 89.0625 89.0625 89.0625
b) Modified divisor 1.4035 3.1438 5.165 6.288
Number of
salespeople assigned 1 3 5 7 16
Learn more about the Jefferson apportionment method at https://brainly.com/question/27954075.
#SPJ1
sasha buys 5 boxes crackers for 12.25 if each box cost the same amount how much does 1 box cost
Sasha buys 5 boxes crackers for 12.25 if each box cost the same amount one box of crackers costs $2.45.
To find the cost of one box of crackers, we can use the concept of unit rate, which is the cost per one unit. In this case, we want to find the cost per one box of crackers.
If Sasha bought 5 boxes of crackers for a total of $12.25, we can set up a proportion to find the cost of one box of crackers:
5 boxes / $12.25 = 1 box / x
Here, "x" represents the cost of one box of crackers. We can solve for "x" by cross-multiplying the proportion:
5 boxes * x = $12.25 * 1 box
5x = $12.25
x = $12.25 / 5
x = $2.45
To check our answer, we can multiply the cost of one box by the number of boxes Sasha bought:
$2.45 * 5 boxes = $12.25
This confirms that our answer is correct.
To learn more about cost click on,
https://brainly.com/question/13067937
#SPJ1
Complete question is:
Sasha buys 5 boxes crackers for 12.25, if each box cost the same amount, then how much does 1 box of the crackers cost?
A law firm is going to designate associates and partners to a big new case. The daily rate charged to the client for each associate is $400 and the daily rate for each partner is $1300. The law firm assigned 2 more associates than partners to the case and was able to charge the client $4200 per day for these lawyers' services. Determine the number of associates assigned to the case and the number of partners assigned to the case.Charlotte runs a farm stand that sells bananas and raspberries. Each pound of bananas sells for $3.25 and each pound of raspberries sells for $1.75. Charlotte sold 13 more pounds of bananas than pounds of raspberries and made $227.25 altogether. Determine the number of pounds of bananas sold and the number of pounds of raspberries sold.
Answer:
1. The number of associates = 4
The number of partners = 2
2. Pounds of bananas sold = 50
Pounds of raspberries sold = 37
Step-by-step explanation:
1. Let,
Number of associates = x
Number of partners = y
According to given statement;
x = y + 2 Eqn 1
400x + 1300y = 4200 Eqn 2
Putting value of x from Eqn 1 in Eqn 2
400(y+2)+1300y=4200
400y+800+1300y=4200
1700y=4200-800
1700y = 3400
Dividing both sides by 1700
\(\frac{1700y}{1700}=\frac{3400}{1700}\\y=2\)
Putting y = 2 in Eqn 1
x = 2 + 2
x = 4
Hence,
The number of associates = 4
The number of partners = 2
2. Let,
b represents the pounds of bananas
r represent the pounds of raspberries
According to given statement,
b = r + 13 Eqn 1
3.25b + 1.75r = 227.25 Eqn 2
Putting value of b from Eqn 1 in Eqn 2
3.25(r+13) + 1.75r = 227.25
3.25r + 42.25 + 1.75r = 227.25
5r = 227.25-42.25
5r = 185
Dividing both sides by 5
\(\frac{5r}{5}=\frac{185}{5}\\r=37\)
Putting r = 37 in Eqn 1
b = 37+13 = 50
Hence,
Pounds of bananas sold = 50
Pounds of raspberries sold = 37
Company A makes a large shipment to Company B. Company B can reject the shipment if they can conclude that the proportion of defective items in the shipment is larger than 0.1. In a sample of 400 items from the shipment, Company B finds that 59 are defective. Conduct the appropriate hypothesis test for Company B using a 0.05 level of significance.
Answer:
\(z=\frac{0.1475-0.1}{\sqrt{\frac{0.1(1-0.1)}{400}}}=3.17\)
The p value for this case would be given by:
\(p_v =P(z>3.17)=0.00076\)
For this case the p value is lower than the significance level so then we have enough evidence to reject the null hypothesis and we can conclude that the true mean is significantly higher than 0.1 and then Company B can reject the shipment
Step-by-step explanation:
Information provided
n=400 represent the random sample taken
X=59 represent number of defectives from the company B
\(\hat p=\frac{59}{400}=0.1475\) estimated proportion of defectives from the company B
\(p_o=0.1\) is the value to verify
\(\alpha=0.05\) represent the significance level
z would represent the statistic
\(p_v\) represent the p value
Hypothesis to test
We want to verify if the true proportion of defectives is higher than 0.1 then the system of hypothesis are.:
Null hypothesis:\(p \leq 0.1\)
Alternative hypothesis:\(p > 0.1\)
The statistic would be given by:
\(z=\frac{\hat p -p_o}{\sqrt{\frac{p_o (1-p_o)}{n}}}\) (1)
Replacing the info given we got:
\(z=\frac{0.1475-0.1}{\sqrt{\frac{0.1(1-0.1)}{400}}}=3.17\)
The p value for this case would be given by:
\(p_v =P(z>3.17)=0.00076\)
For this case the p value is lower than the significance level so then we have enough evidence to reject the null hypothesis and we can conclude that the true mean is significantly higher than 0.1 and then Company B can reject the shipment
Tiffany had finished 78% of her homework. Write this amount as a decimal.
Could someone please help me!?
A cylindrical pillers has a diameter of 56 cm and is of 35 m high there are 16 pillers around the building find the cost of painting the curved surface area of all the pillars at the rate of 5.50 per 1m square
Answer:
Answer:
Total cuŕved surface area of the pillars = 985.6m²Total cost of painting = Rs.5420.8
Explanation:
Given,
A cylindrical pillar with dimènsions,
r(radius) = 56/2 = 28cm. [ Since d = r/2]h(height) = 35m
There are 16 such pillars. Find the cost of paiñting the the curved surface area of all pillars 5.50/m²
Conversion of the dimensioñs in same units.
Radius = 28cm
= 28 ÷ 100 [Since 1m = 100]
= 0.28m
Cuŕved surface area of the pillar is given by,
→ 2πrh
Where,
Taking ‘π’ as 22/7
→ 2*(22/7)*(0.28)*(35)